Вы находитесь на странице: 1из 26

COMPILATION OF CASE DIGEST CONSTI 2: LIBERTY OF ABODE AND TRAVEL AND FREEDOM OF ASSOCIATION

ISV Notes

LIBERTY OF ABODE
1. SALONGA V. HERMOSO
97 SCRA 121 Political Law Right to Travel Even
During Martial Law
FACTS:
This is not the first time Jovito R. Salonga came to the
Supreme Court by way of a mandamus proceeding to
compel the issuance to him of a certificate of eligibility to
travel. In the first case, Salonga v. Madella (GR L49130), the case became moot and academic as the
Office of the Solicitor General, in its answer to the
petition, stated that the travel eligibility certificate was not
denied and, as a matter of fact, had been granted.
Herein, in the motion to dismiss of the Solicitor General
dated 21 April 1980, it was stated that the certificate of
eligibility to travel had been granted Salonga. A xeroxed
copy was enclosed.
ISSUE: Whether or not the right to travel may be
prohibited during martial law.

HELD:
The Travel Processing Center should exercise the
utmost care to avoid the impression that certain
citizens desirous of exercising their constitutional
right to travel could be subjected to inconvenience
or annoyance. In the address of President and Prime
Minister Ferdinand E. Marcos before the American
Newspaper Publishers Association on 22 April 1980, he
emphasized anew the respect accorded constitutional
rights. The freedom to travel is certainly one of the most
cherished. He cited with approval the ringing affirmation
of Willoughby, who, as he noted was "partial to the
claims of liberty." Burdick and Willis, both of whom were
equally convinced that there be no erosion to human
rights even in times of martial law, likewise received from
President Marcos the accolade of his approval. It would
appear, therefore, that in case of doubt of the Officer-inCharge of the Travel Processing Center, the view of
General Fabian Ver should immediately be sought. It
goes without saying that the petition for such certificate
of eligibility to travel be filed at the
earliest opportunity to facilitate the granting thereof and
preclude any disclaimer as to the person desiring to
travel being in any way responsible for any delay.

2.

MARCOS VS. MANGLAPUS [GR 88211, 15


September 1989]
En Banc, Cortes (J): 6 concur, 1 concurs in separate
opinion, 5 dissent in separate opinions, 1 on leave
FACTS:

In February 1986, Ferdinand E. Marcos was deposed


from the presidency via the non-violent "people
power" revolution and forced into exile. In his stead,
Corazon C. Aquino was declared President of the
Republic under a revolutionary government. Her
ascension to and consolidation of power have not been
unchallenged. The failed Manila Hotel coup in 1986 led
by political leaders of Mr. Marcos, the takeover of
television station Channel 7 by rebel troops led by Col.
Canlas with the support of "Marcos loyalists" and the
unsuccessful plot of the Marcos spouses to
surreptitiously return from Hawaii with mercenaries
aboard an
aircraft chartered by a Lebanese arms dealer awakened
the nation to the capacity of the Marcoses to stir
trouble even from afar and to the fanaticism and blind
loyalty of their followers in the country. The
ratification of the 1987 Constitution enshrined the victory
of "people power" and also clearly reinforced the
constitutional moorings of Mrs. Aquino's presidency. This
did not, however, stop bloody challenges to the
government. On 28 August 1987, Col. Gregorio
Honasan, one of the major players in the February
Revolution, led a failed coup that left scores of people,
both combatants and civilians, dead. There were
several other armed sorties of lesser significance, but
the message they conveyed was the same a split in
the ranks of the military establishment that threatened
civilian supremacy over the military and brought to the
fore the realization that civilian government could be at
the mercy of a fractious military. But the armed
threats to the Government were not only found in
misguided elements in the military establishment and
among rabid followers of Mr. Marcos. There were also
the communist insurgency and the secessionist
movement in Mindanao which gained ground during the
rule of Mr. Marcos, to the extent that the communists
have set up a parallel government of their own in the
areas they effectively control while the separatists are
virtually free to move about in armed bands. There has
been no let up in these groups' determination to wrest
power from the government. Not only through resort to
arms but also through the use of propaganda have
they been successful in creating chaos and destabilizing
the country. Nor are the woes of the Republic purely
political. The accumulated foreign debt and the plunder
of the nation attributed to Mr. Marcos and his cronies
left the economy devastated. The efforts at economic
recovery, three years after Mrs. Aquino assumed office,
have yet to show concrete results in alleviating the
poverty of the masses, while the recovery of the illgotten
wealth of the Marcoses has remained elusive. Now, Mr.
Marcos, in his deathbed, has signified his wish to
return to the Philippines to die. But Mrs. Aquino,
considering the dire consequences to the nation of his
return
at a time when the stability of government is threatened
from various directions and the economy is just

COMPILATION OF CASE DIGEST CONSTI 2: LIBERTY OF ABODE AND TRAVEL AND FREEDOM OF ASSOCIATION
ISV Notes

beginning to rise and move forward, has stood firmly on


the decision to bar the return of Mr. Marcos and his
family. Hence, Marcos' petition for mandamus and
prohibition, asking the court to order Raul Manglapus as
Secretary of Foreign Affairs, Catalino Macaraig as
Executive Secretary, Sedfrey Ordonez as Secretary of
Justice, Miriam Defensor Santiago as Immigration
Commissioner, Fidel Ramos as Secretary of National
Defense, and Renato de Villa as Chief of Staff, to issue
travel documents to Mr. Marcos and the immediate
members of his family and to enjoin the implementation
of the President's decision to bar their return to the
Philippines.
ISSUE:
Whether Ferdinand E. Marcos and his family have the
right to travel and liberty of abode, in light of the
attendant circumstances in the present case.
HELD:
The individual right involved is not the right to travel from
the Philippines to other countries or within
the Philippines. These are what the right to travel would
normally connote. Essentially, the right involved is
the right to return to one's country, a totally distinct right
under international law, independent from although
related to the right to travel. Thus, the Universal
Declaration of Humans Rights and the International
Covenant on Civil and Political Rights treat the right to
freedom of movement and abode within the territory
of a state, the right to leave a country, and the right to
enter one's country as separate and distinct rights. The
Declaration speaks of the "right to freedom of movement
and residence within the borders of each state"
separately from the "right to leave any country, including
his own, and to return to his country." On the other
hand, the Covenant guarantees the "right to liberty of
movement and freedom to choose his residence" and
the
right to "be free to leave any country, including his own."
which rights may be restricted by such laws as "are
necessary to protect national security, public order,
public health or morals or the separate rights and
freedoms
of others." as distinguished from the "right to enter his
own country" of which one cannot be "arbitrarily
deprived." It would therefore be inappropriate to construe
the limitations to the right to return to one's country
in the same context as those pertaining to the liberty of
abode and the right to travel. The right to return to
one's country is not among the rights specifically
guaranteed in the Bill of Rights, which treats only of the
liberty of abode and the right to travel, but the right to
return may be considered, as a generally accepted
principle of international law and, under our Constitution,
is part of the law of the land. However, it is distinct
and separate from the right to travel and enjoys a
different protection under the International Covenant of
Civil and Political Rights, i.e., against being "arbitrarily
deprived" thereof. On the other hand, the Constitution

declares among the guiding principles that "the prime


duty of the Government is to serve and protect the
people" and that "the maintenance of peace and order,
the protection of life, liberty, and property, and the
promotion of the general welfare are essential for the
enjoyment by all the people of the blessings of
democracy." Admittedly, service and protection of the
people, the maintenance of peace and order, the
protection of life, liberty and property, and the promotion
of the general welfare are essentially ideals to guide
governmental action. Thus, in the exercise of
presidential functions, in drawing a plan of government,
and in
directing implementing action for these plans, or from
another point of view, in making any decision as
President of the Republic, the President has to consider
these principles, among other things, and adhere to
them. Faced with the problem of whether or not the time
is right to allow the Marcoses to return to the
Philippines, the President is, under the Constitution,
constrained to consider these basic principles in arriving
at a decision. More than that, having sworn to defend
and uphold the Constitution, the President has the
obligation under the Constitution to protect the people,
promote their welfare and advance the national
interest. Since the persons who seek to return to the
country are the deposed dictator and his family at whose
door the travails of the country are laid and from whom
billions of dollars believed to be ill-gotten wealth are
sought to be recovered, the constitutional guarantees
must be adjusted to the requirements of equally
important public interests, as such are neither absolute
nor inflexible. The President has determined that the
destabilization caused by the return of the Marcoses
would wipe away the gains achieved during the past few
years and lead to total economic collapse. Given what is
within our individual and common knowledge of the
state of the economy, the Court cannot argue with that
determination.
3.

SILVERIO VS. COURT OF APPEALS [GR


94284, 8 April 1991]

Second Division, Melencio-Herrera (J): 4 concur


Facts:
On 14 October 1985, Ricardo C. Silverio was charged
with violation of Section 20 (4) of the Revised
Securities Act in Criminal Case CBU-6304 of the
Regional Trial Court of Cebu. In due time, he posted bail
for his provisional liberty. On 26 January 1988, or more
than 2 years after the filing of the Information, the
People of the Philippines filed an Urgent ex parte Motion
to cancel the passport of and to issue a holddeparture Order against Silverio on the ground that he
had gone abroad several times without the necessary
Court approval resulting in postponements of the
arraignment and scheduled hearings. Overruling
opposition,

COMPILATION OF CASE DIGEST CONSTI 2: LIBERTY OF ABODE AND TRAVEL AND FREEDOM OF ASSOCIATION
ISV Notes

the Regional Trial Court, on 4 April 1988, issued an


Order directing the Department of Foreign Affairs to
cancel Silverio's passport or to deny his application
therefor, and the Commission on Immigration to prevent
Silverio from leaving the country. This order was based
primarily on the Trial Court's finding that since the
filing of the Information on 14 October 1985, "the
accused has not yet been arraigned because he has
never
appeared in Court on the dates scheduled for his
arraignment and there is evidence to show that accused
Ricardo C. Silverio, Sr. has left the country and has gone
abroad without the knowledge and permission of
this Court." Silverio's Motion for Reconsideration was
denied on 28 July 1988. Silverio's Certiorari Petition
before the Court of Appeals met a similar fate on 31
January 1990. Hence, the Petition for Review filed on 30
July 1990.
Issue:
Whether the right to travel can be impaired upon lawful
order of the Court, even on grounds other than
the "interest of national security, public safety or public
health."
Held:
Article III, Section 6 of the 1987 Constitution should be
interpreted to mean that while the liberty of
travel may be impaired even without Court Order, the
appropriate executive officers or administrative
authorities are not armed with arbitrary discretion to
impose limitations. They can impose limits only on the
basis of "national security, public safety, or public health"
and "as may be provided by law," a limitive phrase
which did not appear in the 1973 text. Apparently, the
phraseology in the 1987 Constitution was a reaction to
the ban on international travel imposed under the
previous regime when there was a Travel Processing
Center,
which issued certificates of eligibility to travel upon
application of an interested party. Article III, Section 6 of
the 1987 Constitution should by no means be construed
as delimiting the inherent power of the Courts to use
all means necessary to carry their orders into effect in
criminal cases pending before them. When by law
jurisdiction is conferred on a Court or judicial officer, all
auxiliary writs, process and other means necessary
to carry it into effect may be employed by such Court or
officer. Herein, Silverio is facing a criminal charge.
He has posted bail but has violated the conditions
thereof by failing to appear before the Court when
required.
Warrants for his arrest have been issued. Those orders
and processes would be rendered nugatory if an
accused were to be allowed to leave or to remain, at his
pleasure, outside the territorial confines of the
country. Holding an accused in a criminal case within the
reach of the Courts by preventing his departure
from the Philippines must be considered as a valid
restriction on his right to travel so that he may be dealt

with in accordance with law. The offended party in any


criminal proceeding is the People of the Philippines. It
is to their best interest that criminal prosecutions should
run their course and proceed to finality without
undue delay, with an accused holding himself amenable
at all times to Court Orders and processes.

4. CAUNCA VS SALAZAR
Facts: This is an action for habeas corpus brought by
Bartolome Caunca in behalf of his cousin Estelita Flores
who was employed by the Far Eastern Employment
Bureau, owned by Julia Salazar, respondent herein. An
advanced payment has already been given to Estelita by
the employment agency, for her to work as a maid.
However, Estelita wanted to transfer to another
residence, which was disallowed by the employment
agency. Further she was detained and her liberty was
restrained. The employment agency wanted that the
advance payment, which was applied to her
transportation expense from the province should be paid
by Estelita before she could be allowed to leave.
Issue: Whether or Not an employment agency has the
right to restrain and detain a maid without returning the
advance payment it gave?
Held: An employment agency, regardless of the amount
it may advance to a prospective employee or maid, has
absolutely no power to curtail her freedom of movement.
The fact that no physical force has been exerted to keep
her in the house of the respondent does not make less
real the deprivation of her personal freedom of
movement, freedom to transfer from one place to
another, freedom to choose ones residence. Freedom
may be lost due to external moral compulsion, to
founded or groundless fear, to erroneous belief in the
existence of an imaginary power of an impostor to cause
harm if not blindly obeyed, to any other psychological
element that may curtail the mental faculty of choice or
the unhampered exercise of the will. If the actual effect
of such psychological spell is to place a person at the
mercy of another, the victim is entitled to the protection
of courts of justice as much as the individual who is
illegally deprived of liberty by duress or physical
coercion.

5. Villavicencio vs Lukban L-14639


Facts:
Justo Lukban as Manila Mayor together with the police
officer, took custody of 170 women at the night of
October 25 beyond the latter's consent and knowledge
and thereafter were shipped to Davao City where they

COMPILATION OF CASE DIGEST CONSTI 2: LIBERTY OF ABODE AND TRAVEL AND FREEDOM OF ASSOCIATION
ISV Notes

were signed as laborers.


A writ of habeas corpus was filed against the mayor on
behalf of those women. The court granted the writ, but
the mayor was not able to bring any of the women
before the court on the stipulated date.
Issue:
Whether or not the act of mayor has a legal basis.
Held:
The supreme court said that the mayor's acts were not
legal. His intent of exterminating vice was
commendable, but there was no law saying that he
could force filipino women to change their domicile from
manila to nother place. The women, said the court,
although in a sense "lepers of society" were still filipino
citizens and such they were entitled to the constitutional
enjoyed by all other filipino citizens. The right to freedom
of domicile was such a fundamental right that its
suppression could considered tantamount to slavery.
The supreme court upheld the right of filipino citizens to
freedom of domicile or the Liberty of abode. "Ours is a
government of laws and not of men."

6. Angel Lorenzo v. The Director of Health


FACTS:
The Petitioner, who suffered from leprosy (Hansens
disease), was confined in the San Lazaro Hospital in
Manila in conformity with 1058 of the Administrative
Code, which empowered the Director of Health and his
authorized agents to cause to be apprehended, and
detained, isolated, or confined, all leprous persons in the
Philippine Islands. The Petitioner brought a petition for
habeas corpus, alleging that 1058 was unconstitutional.
The Court of First Instance of Manila sustained 1058 and
denied the petition for habeas corpus. The
petitioner appealed this judgment, arguing that the trial
court was required to receive evidence to determine
whether or not leprosy was a contagious disease.
HELD:
The Court held that Article XV of Chapter 37, 1058 of the
Administrative Code was constitutional. Section
1058 of the Administrative Code was a legitimate
exercise of a police power that extended to the
protection
of the public health and leprosy undoubtedly qualified as
a serious public health problem. Leprosy was
commonly believed to be an infectious disease and
scientific authority supported segregation of lepers as a
means of preventing the spread of the disease. The local
legislature had regarded leprosy as a contagious
disease and had authorized lawful measures to control
its spread. Whether or not this determination was

correct, it had been decided according to due process of


law, and was not open to the Court to review.
Decision Excerpts
The methods provided for the control of leprosy plainly
constitute due process of law. The assumption must
be that if evidence was required to establish the
necessity for the law, that it was before the legislature
when
the act was passed. In the case of a statute purporting
the have been enacted in the interest of the public
health, all questions relating to the determination of
matters of fact are for the legislature. If there is probable
basis for sustaining the conclusion reached, its findings
are not subject to judicial review. Debatable
questions are for the Legislature to decide. The courts
do not sit to resolve the merits of conflicting theories.
Similarly, the local legislature has regarded leprosy as a
contagious disease and has authorized measures to
control the dread scourge. To that forum must the
petitioner go to reopen the question. We are frank to say
that it would require a much stronger case than the one
at bar for us to sanction admitting the testimony of
expert or other witnesses to show that a law of this
character may possibly violate some constitutional
provision.

FREEDOM OF
INFORMATION
1. RE: REQUEST FOR COPY OF 2008 STATEMENT
OF ASSETS, LIABLITIES AND NETWORTH (SALN)
AND PERSONAL DATA SHEET OR CURRICULUM
VITAE OF THE JUSTICES OF THE SUPREME COURT
AND OFFICERS AND EMPLOYEES OF THE
JUDICIARY AM No. 09-8-6- SC, 13 June 2012, EN
BANC (Mendoza, J.)

[UST Law Review Vol LVII, No. 1, NOVEMBER


2012]

In accordance with section 17, Article IX of the


1987 Philippine Constitution, there is a duty on the part
of the members of the judiciary to disclose their SALNs
and other personal documents, being a matter of public
concern and interest, as long as it is made in a manner
provided by law.
The Court received two letters from Rowena C.
Paraan and Karol M. Ilagan requesting for copies of the
Statement of Assets, Liabilities and Networth (SALN)
and Personal Data Sheet (PDS) or the Curriculum Vitae
(CV) of its Justices for the year 2008 for the purpose of
updating their database of information on government
officials. The special committee created by the Supreme
Court (SC) to review the request issued a Memorandum
recommending the creation of a Committee on Public

COMPILATION OF CASE DIGEST CONSTI 2: LIBERTY OF ABODE AND TRAVEL AND FREEDOM OF ASSOCIATION
ISV Notes

Disclosure to take over the functions of the Office of the


Court Administrator (OCA) with respect to requests for
copies of SALN and other personal documents of
members of the Judiciary. Meanwhile, several and other
personal documents of the Justices of the Court, the
Court of Appeals (CA) and the Sandigan Bayan (SB)
were filed.
ISSUE:
Whether or not the Supreme Court Justices are
mandated by law to release their SALN to the public
HELD:
Section 7 of Article III of the Constitution is
relevant in the issue of public disclosure of SALN and
other documents of public officials, which provides that
the right of the people to information on matters of public
concern shall be recognized. Access to official records,
and to documents, and papers pertaining to official acts,
transactions, or decisions, as well as to government
research data used as basis for policy development,
shall be afforded the citizen, subject to such limitations
as may be provided by law.
Emphasizing the import and meaning of the
foregoing constitutional provision, the Court in the
landmark case of Valmonte v. Belmonte, Jr., elucidated
that the right to information goes hand-in-hand with the
constitutional policies of full public disclosure and
honesty in the public service. It is meant to enhance the
widening role of the citizenry in governmental decision
making as well as in checking abuse in government. The
importance of the said right was pragmatically explicated
that the incorporation of this right in the Constitution is a
recognition of the fundamental role of free exchange of
information in a democracy. There can be no realistic
perception by the public of the nations problems, nor a
meaningful democratic decision-making if they are
denied access to information of general interest.
Information is needed to enable them members of
society to cope with the exigencies of the times.
However, restrictions on the access to certain records
may be imposed by law.
Thus, while public concern like public interest
eludes exact definition and has been said to embrace a
broad spectrum of subjects which the public may want to
know, either because such matters directly affect their
lives, or simply because such matters naturally arouse
the interest of an ordinary citizen, the Constitution itself,
under Section 17, Article XI, has classified the
information disclosed in the SALN as a matter of public
concern and interest. In other words, a duty to disclose
sprang from the right to know. Both of constitutional
origin, the former is a command while the latter is a
permission. Hence, there is a duty on the part of
members of the government to disclose their SALNs to
the public in the manner provided by law.

In the case at bar, the Court notes the valid


concerns of the other magistrates regarding the possible
illicit motives of some individuals in their request for
access to such personal information and their
publication. However, custodians of public documents
must not concern themselves with motives, reasons and
objects of the persons seeking access to the records.
The moral or material injury which their misuse might
inflict on others is the requestors responsibility and
lookout. While public officers in the custody or control of
public records have the discretion to regulate the
manner in which records may be inspected, examined or
copied by interested persons, such discretion does not
carry with the authority to prohibit access, inspection,
examination or copying of the records. After all, public
office is a public trust

2. Valmonte Vs Belmonte
FACTS : Petitioners in this special civil action for
mandamus with preliminary injunction invoke their right
to information and pray that respondent be directed: (a)
to furnish petitioners the list of the names of the
Batasang Pambansa members belonging to the UNIDO
and PDP-Laban who were able to secure clean loans
immediately before the February 7 election thru the
intercession/marginal note of the then First Lady Imelda
Marcos; and/or (b) to furnish petitioners with certified
true copies of the documents evidencing their respective
loans; and/or (c) to allow petitioners access to the public
records for the subject information On June 20, 1986,
apparently not having yet received the reply of the
Government Service and Insurance System (GSIS)
Deputy General Counsel, petitioner Valmonte wrote
respondent another letter, saying that for failure to
receive a reply, "(W)e are now considering ourselves
free to do whatever action necessary within the premises
to pursue our desired objective in pursuance of public
interest."
ISSUE : WON Valmonte, et. al. are entitled as citizens
and taxpayers to inquire upon GSIS records on behest
loans given by the former First Lady Imelda Marcos to
Batasang Pambansa members belonging to the UNIDO
and PDP-Laban political parties.
HELD : Respondent has failed to cite any law granting
the GSIS the privilege of confidentiality as regards the
documents subject of this petition. His position is
apparently based merely on considerations of policy. The
judiciary does not settle policy issues. The Court can
only declare what the law is, and not what the law should
be. Under our system of government, policy issues are
within the domain of the political branches of the
government, and of the people themselves as the
repository of all State power. The concerned borrowers
themselves may not succeed if they choose to invoke
their right to privacy, considering the public offices they
were holding at the time the loans were alleged to have

COMPILATION OF CASE DIGEST CONSTI 2: LIBERTY OF ABODE AND TRAVEL AND FREEDOM OF ASSOCIATION
ISV Notes

been granted. It cannot be denied that because of the


interest they generate and their newsworthiness, public
figures, most especially those holding responsible
positions in government, enjoy a more limited right to
privacy as compared to ordinary individuals, their actions
being subject to closer public scrutiny The "transactions"
used here I suppose is generic and, therefore, it can
cover both steps leading to a contract, and already a
consummated contract, Considering the intent of the
framers of the Constitution which, though not binding
upon the Court, are nevertheless persuasive, and
considering further that government-owned and
controlled corporations, whether performing proprietary
or governmental functions are accountable to the
people, the Court is convinced that transactions entered
into by the GSIS, a government-controlled corporation
created by special legislation are within the ambit of the
people's right to be informed pursuant to the
constitutional policy of transparency in government
dealings. Although citizens are afforded the right to
information and, pursuant thereto, are entitled to "access
to official records," the Constitution does not accord
them a right to compel custodians of official records to
prepare lists, abstracts, summaries and the like in their
desire to acquire information on matters of public
concern.

3. Legaspi vs. CSC G.R. No. L-72119, May 29, 1987


Facts: The respondent CSC had denied petitioner
Valentin Legaspis request for information on the civil
service eligibilities of Julian Sibonghanoy and Mariano
Agas who were employed as sanitarians in the Health
Department of Cebu City. Sibonghanoy and Agas had
allegedly represented themselves as civil service
eligibles who passed the civil service examinations for
sanitarians.
Claiming that his right to be informed of the eligibilities of
Sibonghanoy and Agas is guaranteed by the
Constitution, and that he has no other plain, speedy and
adequate remedy to acquire the information, petitioner
prays for the issuance of the extraordinary writ of
mandamus to compel the respondent CSC to disclose
said information.
The respondent CSC takes issue on the personality of
the petitioner to bring the suit. It is asserted that the
petition is bereft of any allegation of Legaspis actual
interest in the civil service eligibilities of Sibonghanoy
and Agas.
Issue: Whether or not the petitioner has legal standing
to bring the suit

Held: The petitioner has firmly anchored his case upon


the right of the people to information on matters of public
concern, which, by its very nature, is a public right. It has
been held in the case of Tanada vs. Tuvera, 136 SCRA
27, that when the question is one of public right and the
object of the mandamus is to procure the enforcement of
a public duty, the people are regarded as the real party
in interest, and the person at whose instigation the
proceedings are instituted need not show that he has
any legal or special interest in the result, it being
sufficient to show that he is a citizen and as such
interested in the execution of the laws.
It becomes apparent that when a mandamus proceeding
involves the assertion of a public right, the requirement
of personal interest is satisfied by the mere fact that the
petitioner is a citizen, and therefore, part of the general
public which possesses the right.
The petitioner, being a citizen who as such, is clothed
with personality to seek redress for the alleged
obstruction of the exercise of the public right.

4.Bantay vs COMELEC
Facts: There were two consolidated petitions for
certiorari and mandamus to nullify and set aside certain
issuances of the Commission on Elections (Comelec)
respecting party-list groups which have manifested their
intention to participate in the party-list elections on May
14, 2007.In the first petition, docketed as G.R. No.
177271, petitioners Bantay Republic Act (BA-RA 7941,
for short) and the Urban Poor for Legal Reforms (UP-LR,
for short) assail the various Comelec resolutions
accrediting private respondents Biyaheng Pinoy et al., to
participate in the forthcoming party-list elections on May
14, 2007 without simultaneously determining whether or
not their respective nominees possess the requisite
qualifications defined in Republic Act (R.A.) No. 7941, or
the "Party-List System Act" and belong to the
marginalized and underrepresented sector each seeks
to represent. In the second, docketed as G.R. No.
177314, petitioners Loreta Ann P. Rosales, Kilosbayan
Foundation and Bantay Katarungan Foundation impugn
Comelec Resolution 07-0724 dated April 3, 2007
effectively denying their request for the release or
disclosure of the names of the nominees of the fourteen
(14)accredited participating party-list groups mentioned
in petitioner Rosales previous letter -request While both
petitions commonly seek to compel the Comelec to
disclose or publish the names of the nominees of the
various party-list groups named in the petitions, BA-RA
7941 and UP-LR have the additional prayers that the 33
private respondents named therein be declare[d] as
unqualified to participate in the party-list elections and
that the Comelec be enjoined from allowing respondent
groups from participating in the elections.
Issues:

COMPILATION OF CASE DIGEST CONSTI 2: LIBERTY OF ABODE AND TRAVEL AND FREEDOM OF ASSOCIATION
ISV Notes

1. Can the Court cancel the accreditation accorded by


the COMELEC to the respondent party-list groups
named in their petition on the ground that these groups
and their respective nominees do not appear to be
qualified?
2. Whether respondent COMELEC, by refusing to reveal
the names of the nominees of the various party-list
groups, has violated the right to information and free
access to documents as guaranteed by the Constitution;
and
3. Whether respondent COMELEC is mandated by the
Constitution to disclose to the public the names of said
nominees.
Ruling:
1. The Court is unable to grant the desired plea of
petitioners BA-RA 7941 and UP-LR for cancellation of
accreditation on the grounds thus advanced in their
petition. The exercise would require the Court to make a
factual determination, a matter which is outside the office
of judicial review by way of special civil action for
certiorari. In certiorari proceedings, the Court is not
called upon to decide factual issues and the case must
be decided on the undisputed facts on record. The sole
function of a writ of certiorari is to address issues of want
of jurisdiction or grave abuse of discretion and does not
include a review of the tribunals evaluation of the
evidence. (note that nowhere in R.A. No. 7941 is there a
requirement that the qualification of a party-list nominee
be determined simultaneously with the accreditation of
an organization. )
2. COMELECs basis of its refusal to disclose the names
of the nominees of subject party
-list groups,
Section 7 of R.A. 7941,which last sentence reads: [T
]he names of the party-list nominees shall not be
shown on the certified list is certainly not a justifying
card for the Comelec to deny the requested
disclosure. There is absolutely nothing in R.A. No. 7941
that prohibits the Comelec from disclosing or even
publishing through mediums other than the Certified
List of the names.
Assayed against the non-disclosure stance of the
Comelec and the given rationale therefor is the right to
information enshrined in the self-executory 15
Section 7, Article III of the Constitution, viz
:Sec.7. The right of the people to information on matters
of public concern shall be recognized. Access to official
records, and to documents, and papers pertaining to
official acts, transactions, or decisions, as well to
government research data used as basis for policy
development, shall be afforded the citizen, subject to
such limitations as may be provided by law.
Complementing and going hand in hand with the right to
information is another constitutional provision
enunciating the policy of full disclosure and transparency
in Government. We refer to Section 28, Article II of the

Constitution reading: Sec. 28. Subject to reasonable


conditions prescribed by law, the State adopts and
implements a policy of full public disclosure of all its
transactions involving public interest. It has been
repeatedly said in various contexts that the people have
the right to elect their representatives on the basis of an
informed judgment. Hence the need for voters to be
informed about matters that have a bearing on their
choice While the vote cast in a party-list elections is a
vote for a party, such vote, in the
end, would be a vote for its nominees, who, in
appropriate cases, would eventually sit in the House of
Representatives. The Court frowns upon any
interpretation of the law or rules that would hinder in
anyway the free and intelligent casting of the votes in an
election.
3.COMELEC has a constitutional duty to disclose and
release the names of the nominees of the party-list
groups named in the herein petitions. The right to
information is a public right where the real parties in
interest are the public, or the citizens to be precise, but
like all constitutional guarantees, however, the right to
information and its companion right of access to official
records are not absolute. The peoples right to know is
limited to matters of public concern and is further
subject to such limitation as may be provided by law. But
no national security or like concerns is involved in the
disclosure of the names of the nominees of the party-list
groups in question. Doubtless, the Comelec committed
grave abuse of discretion in refusing the legitimate
demands of the petitioners for a list of the nominees of
the party-list groups subject of their respective petitions.
The 1st petition is partly DENIED insofar as it seeks to
nullify the accreditation of the respondents named
therein. However, insofar as it seeks to compel the
Comelec to disclose or publish the names of the
nominees of party-list groups, sectors or organizations
accredited to participate in the May 14, 2007elections,
the 2 petitions are GRANTED. Accordingly, the Comelec
is hereby ORDERED to immediately disclose and
release the names of the nominees of the party-list
groups.

5. Echegaray vs Secretary of Justice


Facts:The Supreme Court affirmed the conviction of
petitioner Leo Echegaray y Pilo for the crime of rape of
the 10 year-old daughter of his common-law spouse.
The supreme penalty of death was to be imposed upon
him. He then filed motion for recon and a supplemental
motion for recon raising constitutionality of Republic Act
No. 7659 and the death penalty for rape. Both were
denied. Consequently, Congress changed the mode of
execution of the death penalty from electrocution to
lethal injection, and passed Republic Act No. 8177,
designating death by lethal injection. Echegaray filed a

COMPILATION OF CASE DIGEST CONSTI 2: LIBERTY OF ABODE AND TRAVEL AND FREEDOM OF ASSOCIATION
ISV Notes

Petition for prohibition from carrying out the lethal


injection against him under the grounds that it
constituted 1. cruel, degrading, or unusual punishment,
2. Being violative of due process, 3. a violation of the
Philippines obligations under international covenants, 4.
an undue delegation of legislative power by Congress,
an unlawful exercise by respondent Secretary of the
power to legislate, and an unlawful delegation of
delegated powers by the Secretary of Justice. In his
motion to amend, the petitioner added equal protection
as a ground.

The Solicitor General stated that the Supreme Court has


already upheld the constitutionality of the Death Penalty
Law, and has declared that the death penalty is not
cruel, unjust, excessive or unusual punishment;
execution by lethal injection, as authorized under R.A.
No. 8177 and the questioned rules, is constitutional,
lethal injection being the most modern, more humane,
more economical, safer and easier to apply (than
electrocution or the gas chamber); in addition to that, the
International Covenant on Civil and Political Rights does
not expressly or impliedly prohibit the imposition of the
death penalty.

Issues: 1. Is the lethal injection a cruel, degrading or


inhuman punishment? 2. Is it a violation of our
international treaty obligations? 3. Is it discriminatory
(pertaining to sec 17)?

Held: 1. No 2. Yes 3rd. Petition denied.

1. Petitioner contends that death by lethal injection


constitutes cruel, degrading and inhuman punishment
because (1) R.A. No. 8177 fails to provide for the drugs
to be used in carrying out lethal injection, the dosage for
each drug to be administered, and the procedure in
administering said drug/s into the accused; (2) its
implementing rules are uncertain as to the date of the
execution, time of notification, the court which will fix the
date of execution, which uncertainties cause the greatest
pain and suffering for the convict; and (3) the possibility
of mistakes in administering the drugs renders lethal
injection inherently cruel. It is well-settled in
jurisprudence that the death penalty per se is not a cruel,
degrading or inhuman punishment. In Harden v. Director
of Prisons- punishments are cruel when they involve

torture or a lingering death; but the punishment of death


is not cruel, within the meaning of that word as used in
the constitution. It implies there something inhuman and
barbarous, something more than the mere
extinguishment of life. Would the lack in particularity
then as to the details involved in the execution by lethal
injection render said law cruel, degrading or inhuman?
The Court believes not. Petitioner contends that Sec. 16
of R.A. No. 8177 is uncertain as to which court will fix
the time and date of execution, and the date of execution
and time of notification of the death convict. As petitioner
already knows, the court which designates the date of
execution is the trial court which convicted the accused.
The procedure is that the judgment is entered fifteen
(15) days after its promulgation, and 10 days thereafter,
the records are remanded to the court below including a
certified copy of the judgment for execution. Neither is
there any uncertainty as to the date of execution nor the
time of notification. As to the date of execution, Section
15 of the implementing rules must be read in conjunction
with the last sentence of Section 1 of R.A. No. 8177
which provides that the death sentence shall be carried
out not earlier than one (1) year nor later then eighteen
(18) months from the time the judgment imposing the
death penalty became final and executory, without
prejudice to the exercise by the President of his
executive clemency powers at all times. Hence, the
death convict is in effect assured of eighteen (18)
months from the time the judgment imposing the death
penalty became final and executor wherein he can seek
executive clemency and attend to all his temporal and
spiritual affairs. Petitioner also contends that the infliction
of wanton pain in case of possible complications in the
intravenous injection that respondent Director is an
untrained and untested person insofar as the choice and
administration of lethal injection is concerned, renders
lethal injection a cruel, degrading and inhuman
punishment. This is unsubstantiated. First. Petitioner has
neither alleged nor presented evidence that lethal
injection required the expertise only of phlebotomists
and not trained personnel and that the drugs to be
administered are unsafe or ineffective. Petitioner simply
cites situations in the United States wherein execution
by lethal injection allegedly resulted in prolonged and
agonizing death for the convict, without any other
evidence whatsoever. Second. Petitioner overlooked
Section 1, third paragraph of R.A. No. 8177 which
requires that all personnel involved in the execution
proceedings should be trained prior to the performance
of such task. We must presume that the public officials
entrusted with the implementation of the death penalty
will carefully avoid inflicting cruel punishment. Third. Any
infliction of pain in lethal injection is merely incidental in

COMPILATION OF CASE DIGEST CONSTI 2: LIBERTY OF ABODE AND TRAVEL AND FREEDOM OF ASSOCIATION
ISV Notes

carrying out the execution of death penalty and does not


fall within the constitutional proscription against cruel,
degrading and inhuman punishment. In a limited sense,
anything is cruel which is calculated to give pain or
distress, and since punishment imports pain or suffering
to the convict, it may be said that all punishments are
cruel. But of course the Constitution does not mean that
crime, for this reason, is to go unpunished. The cruelty
against which the Constitution protects a convicted man
is cruelty inherent in the method of punishment, not the
necessary suffering involved in any method employed to
extinguish life humanely.

2. Violation of international treaties? In countries which


have not abolished the death penalty, sentence of death
may be imposed only for the most serious crimes in
accordance with the law in force at the time of the
commission of the crime and not contrary to the
provisions of the present Covenant and to the
Convention on the Prevention and Punishment of the
Crime of Genocide. This penalty can only be carried out
pursuant to a final judgment rendered by a competent
court. The punishment was subject to the limitation that it
be imposed for the most serious crimes. Included with
the declaration was the Second Optional Protocol to the
International Covenant on Civil and Political Rights,
Aiming at the Abolition of the Death Penalty was adopted
by the General Assembly on December 15, 1989. The
Philippines neither signed nor ratified said document.

3. Petitioner contends that Section 17 of the


Implementing Rules is unconstitutional for being
discriminatory. SEC. 17. SUSPENSION OF THE
EXECUTION OF THE DEATH SENTENCE. Execution
by lethal injection shall not be inflicted upon a woman
within the three years next following the date of the
sentence or while she is pregnant, nor upon any person
over seventy (70) years of age. In this latter case, the
death penalty shall be commuted to the penalty of
reclusion perpetua with the accessory penalties provided
in Article 40 of the Revised Penal Code. Petitioner
contends that Section 17 amends the instances when
lethal injection may be suspended, without an express
amendment of Article 83 of the Revised Penal Code, as
amended by section 25 of R.A. No. 7659, stating that the
death sentence shall not be inflicted upon a woman
while she is pregnant or within one (1) year after
delivery, nor upon any person over seventy years of age.
While Article 83 of the Revised Penal Code, as amended
by Section 25 of Republic Act No. 7659, suspends the

implementation of the death penalty while a woman is


pregnant or within one (1) year after delivery, Section 17
of the implementing rules omits the one (1) year period
following delivery as an instance when the death
sentence is suspended, and adds a ground for
suspension of sentence no longer found under Article 83
of the Revised Penal Code as amended, which is the
three-year reprieve after a woman is sentenced. This
addition is, in petitioners view, tantamount to a genderbased discrimination. Being an implementing rule,
Section 17 must not override, but instead remain
consistent and in harmony with the law it seeks to
implement.

FREEDOM OF
ASSOCIATION
1. SSS Employees Assoc. v CA
Facts: On June 11, 1987, the SSS filed with the
Regional Trial Court of Quezon City a complaint for
damages with a prayer for a writ of preliminary injunction
against petitioners, alleging that on June 9, 1987, the
officers and members of SSSEA staged an illegal strike
and baricaded the entrances to the SSS Building,
preventing non-striking employees from reporting for
work and SSS members from transacting business with
the SSS; that the strike was reported to the Public
Sector Labor - Management Council, which ordered the
strikers to return to work; that the strikers refused to
return to work; and that the SSS suffered damages as a
result of the strike. The complaint prayed that a writ of
preliminary injunction be issued to enjoin the strike and
that the strikers be ordered to return to work; that the
defendants (petitioners herein) be ordered to pay
damages; and that the strike be declared illegal.

It appears that the SSSEA went on strike after the SSS


failed to act on the union's demands, which included:
implementation of the provisions of the old SSS-SSSEA
collective bargaining agreement (CBA) on check-off of
union dues; payment of accrued overtime pay, night
differential pay and holiday pay; conversion of temporary
or contractual employees with six (6) months or more of
service into regular and permanent employees and their
entitlement to the same salaries, allowances and
benefits given to other regular employees of the SSS;

COMPILATION OF CASE DIGEST CONSTI 2: LIBERTY OF ABODE AND TRAVEL AND FREEDOM OF ASSOCIATION
ISV Notes

and payment of the children's allowance of P30.00, and


after the SSS deducted certain amounts from the
salaries of the employees and allegedly committed acts
of discrimination and unfair labor practices.

The three Resolutions were:

Issue: Whether or not employees of the Social Security


System (SSS) have the right to strike.

2) Resolution No. 2 dealing with the Presidency, the


Prime Minister and the Cabinet, and the National
Assembly; and

Held: The 1987 Constitution, in the Article on Social


Justice and Human Rights, provides that the State "shall
guarantee the rights of all workers to self-organization,
collective bargaining and negotiations, and peaceful
concerted activities, including the right to strike in
accordance with law" [Art. XIII, Sec. 31].
Resort to the intent of the framers of the organic law
becomes helpful in understanding the meaning of these
provisions. A reading of the proceedings of the
Constitutional Commission that drafted the 1987
Constitution would show that in recognizing the right of
government employees to organize, the commissioners
intended to limit the right to the formation of unions or
associations only, without including the right to strike.

Considering that under the 1987 Constitution "the civil


service embraces all branches, subdivisions,
instrumentalities, and agencies of the Government,
including government-owned or controlled corporations
with original charters" [Art. IX(B), Sec. .2(l) see also Sec.
1 of E.O. No. 180 where the employees in the civil
service are denominated as "government employees"]
and that the SSS is one such government-controlled
corporation with an original charter, having been created
under R.A. No. 1161, its employees are part of the civil
service [NASECO v. NLRC, G.R. Nos. 69870 & 70295,
November 24,1988] and are covered by the Civil Service
Commission's memorandum prohibiting strikes. This
being the case, the strike staged by the employees of
the SSS was illegal.

2. Occena v. COMELEC
Facts: The challenge in these two prohibition
proceedings against the validity of three Batasang
Pambansa Resolutions proposing constitutional
amendments goes further than merely assailing their
alleged constitutional infirmity. The rather unorthodox
aspect of these petitions is the assertion that the 1973
Constitution is not the fundamental law.

1) Resolution No. 1 proposing an amendment allowing a


natural-born citizen of the Philippines naturalized in a
foreign country to own a limited area of land for
residential purposes

3) Resolution No. 3 on the amendment to the Article on


the Commission on Elections. The three resolutions
were approved by the Interim
Batasang Pambansa sitting as a constituent assembly
on February 5 and 27, 1981which the date of plebiscite
has been set on April 7, 1981. It is thus within the 90-day
period provided by the Constitution.
Issues:
(1) Whether or not the 1973 Constitution is a
fundamental law.
(2) Whether or not the Interim BatasangPambansa has
the power to propose amendments.
(3) Whether or not the three-fourth votes is necessary to
propose amendments as wellas the standard for proper
submission.
(4) Whether or not the three BatasangPambansa
Resolutions proposing constitutionalamendments are
valid.
Held:
Yes, the Interim Batasang Pambansa has the power
and privilege to propose amendments. On January 17,
1973, the present Constitution came into force and
effect. With such a pronouncement by the Supreme
Court and with the recognition of the cardinal postulate
that what the Supreme Court says is not only entitled to
respect but must also be obeyed, a factor
for instability was removed. Thereafter, as a matter of
law, all doubts were resolved. The 1973Constitution is
the fundamental law. The existence of this power is
indubitable as the applicable
provision in the 1976 Amendments is quite explicit.
The Interim Batasang Pambansa, sitting as a constituent body,
can propose amendments. In that capacity, only a
majority vote is needed. It would be an indefensible
proposition to assert that the three-fourth votes required
when it sits as a legislative body applies as well when it
has been convened as the agency through which
amendments could be proposed. That is not
a requirement as far as a constitutional convention is
concerned. It is not a requirement either when, as in this
case, the Interim Batasang Pambansa exercises its
constituent power to propose amendments. Resolution
No. 1 proposing an amendment allowing a natural-born
citizen of the Philippines naturalized in a foreign country
to own a limited area of land for residential purposes

COMPILATION OF CASE DIGEST CONSTI 2: LIBERTY OF ABODE AND TRAVEL AND FREEDOM OF ASSOCIATION
ISV Notes

was approved by the vote of 122 to 5; Resolution No. 2


dealing with the Presidency, the Prime Minister and the
Cabinet, and the National Assembly by a vote of 147 to
5 with 1 abstention; and Resolution No. 3 on the
amendment to the Article on the Commission on
Elections by a vote of 148 to 2 with 1 abstention The
three resolutions were approved by the Interim
Batasang Pambansa sitting as a constituent assembly
on February 5 and 27, 1981,thus making them valid.

standing were to be issued stickers for use in their


vehicles. Thereafter, on three separate incidents, Victor
M. Gaston, the son of the spouses Gaston who lives with
them, was required by the guards on duty employed by
SCHA to show his driver's license as a prerequisite to his
entrance to the subdivision and to his residence therein
despite their knowing him personally and the exact
location of his residence.

3. In Re Edillon

On 29 March 1998, Victor Ma. Gaston was himself


prevented from entering the subdivision and proceeding
to his residential abode when security guards Roger
Capillo and a "John Doe" lowered the steel bar of the
KAMETAL gate of the subdivision and demanded from
him his driver's license for identification. On 1 April 1998,
Spouses Victor Ma. Gaston and Lydia M. Gaston filed a
complaint for damages with preliminary
injunction/preliminary mandatory injunction and
temporary restraining order before the Regional Trial
Court in Negros Occidental at Bacolod City against
Santa Clara Homeowners Association (SCHA) thru its
Board of Directors, namely: Arneil Chua, Luis Sarrosa,
Jocelyn Garcia, Ma. Milagros Vargas, Lorenzo Lacson,
Ernesto Piccio, Dindo Ilagan, Danilo Gamboa, Jr., Rizza
de la Rama and Security Guard Capillo and 'John Doe',
and Santa Clara Estate, Incorporated (Civil Case 9810217, RTC-Branch 49, Bacolod City); alleging that the
acts of SCHA, et al., done in the presence of other
subdivision owners had caused the spouses Gaston to
suffer moral damage. On 8 April 1998, SCHA, et al. filed
a motion to dismiss arguing that the trial court had no
jurisdiction over the case as it involved an intracorporate dispute between SCHA and its members
pursuant to Republic Act 580, as amended by Executive
Orders 535 and 90, much less, to declare as null and
void the subject resolution of the board of directors of
SCHA, the proper forum being the Home insurance and
Guaranty Corporation (HIGC). To support their claim of
intra-corporate controversy, SCHA, et al. stated that the
Articles of Incorporation of SCHA, which was duly
approved by the Securities and Exchange Commission
(SEC) on 4 October 1973, provides "that the association
shall be a non-stock corporation with all homeowners of
Sta. Clara constituting its membership"; and that its bylaws contains a provision that "all real estate owners in
Sta. Clara Subdivision automatically become members
of the association"; among others. On 6 July 1998, the
lower court resolved to deny SCHA et al.'s motion to
dismiss, finding that there existed no intra-corporate
controversy since the Spouses Gaston alleged that they
had never joined the association.

Facts: This is an administrative case against Edillon


who refuses to pay his IBP membership dues assailing
the provisions of the Rule of Court 139-A and the
provisions of par. 2, Section 24, Article III, of the IBP ByLaws pertaining to the organization of IBP, payment of
membership fee and suspension for failure to pay the
same. He contends that the stated provisions constitute
an invasion of his constitutional rights of being
compelled to be a member of the IBP in order to practice
his profession and thus deprives his rights to liberty and
property and thereby null and void.
Issue: Whether or not it assailed provisions constitutes a
deprivation of liberty and property of the respondent.
Held: The court held that the IBP is a State-organized
Bar as distinguished from bar associations that are
organized by individual lawyers themselves,
membership of which is voluntary. The IBP however is
an official national body of which all lawyers must be a
member and are subjected to the rules prescribed for the
governance of the Bar which includes payment
of reasonable annual fee for the purpose of carrying out
its objectives and implementation of regulations in the
practice of law. The provisions assailed does not infringe
the constitutional rights of the respondent as it is a valid
exercise of police power necessary to perpetuate its
existence with regulatory measures to implement. The
name of Edillon was stricken out from the rolls of
attorney for being a delinquent member of the bar.

4. Sta. Clara Homeowners Association v. Gaston


Facts: Spouses Victor Ma. Gaston and Lydia M. Gaston
were residents of San Jose Avenue, Sta. Clara
Subdivision, Mandalagan, Bacolod City. They purchased
their lots in the said subdivision sometime in 1974, and
at the time of purchase, there was no mention or
requirement of membership in any homeowners'
association. From that time on, they have remained nonmembers of SCHA. They also stated that an
arrangement was made wherein homeowners who were
non-members of the association were issued "nonmember" gatepass stickers for their vehicles for
identification by the security guards manning the
subdivision's entrances and exits. This arrangement
remained undisturbed until sometime in the middle of
March 1998, when SCHA disseminated a board
resolution which decreed that only its members in good

On 18 July 1998, SCHA, et al. submitted a Motion for


Reconsideration, adding lack of cause of action as
ground for the dismissal of the case. On 17 August 1998,
the trial court denied the said motion without however
ruling on the additional ground of lack of cause of action.
On 18 August 1998, SCHA, et al. filed a motion to
resolve its motion to dismiss on ground of lack of cause
of action. On 8 September 1998, the trial court issued an

COMPILATION OF CASE DIGEST CONSTI 2: LIBERTY OF ABODE AND TRAVEL AND FREEDOM OF ASSOCIATION
ISV Notes

order denying the motion. On 24 September 1998,


SCHA. et al. elevated the matter to the Court of Appeals
via a Petition for Certiorari. On 31 August 1999, the
Court of Appeals dismissed the Petition and ruled that
the RTC had jurisdiction over the dispute. The appellate
court likewise denied SCHA, et al.'s motion for
reconsideration in a resolution dated 11 February 2000.
SCHA, et al. filed the petition for review.
Issue: Whether the Spouses Gaston are members of the
SCHA.
Held: The constitutionally guaranteed freedom of
association includes the freedom not to associate. The
right to choose with whom one will associate oneself is
the very foundation and essence of that partnership.
Further, the Spouses Gaston cannot be compelled to
become members of the SCHA by the simple expedient
of including them in its Articles of Incorporation and Bylaws without their express or implied consent. True, it
may be to the mutual advantage of lot owners in a
subdivision to band themselves together to promote their
common welfare, but that is possible only if the owners
voluntarily agree, directly or indirectly, to become
members of the association. True also, memberships in
homeowners' associations may be acquired in various
ways often through deeds of sale, Torrens certificates
or other forms of evidence of property ownership.
Herein, however, other than the said Articles of
Incorporation and By-laws, there is no showing that the
Spouses Gaston have agreed to be SCHA members.
The approval by the SEC of the said documents is not
an operative act which bestows membership on the
Spouses Gaston because the right to associate partakes
of the nature of freedom of contract which can be
exercised by and between the homeowners amongst
themselves, the homeowners' association and a
homeowner, and the subdivision owner and a
homeowner/lot buyer. Clearly, there is no privity of
contract exists between SCHA and Spouses Gaston.
When the Spouses Gaston purchased their property in
1974 and obtained Transfer Certificates of Titles T126542 and T-127462 for Lots 11 and 12 of Block 37
along San Jose Avenue in Sta. Clara Subdivision, there
was no annotation showing their automatic membership
in the SCHA. Furthermore, the records are bereft of any
evidence that would indicate that the Spouses Gaston
intended to become members of the SCHA. Prior to the
implementation of the aforesaid Resolution, they and the
other homeowners who were not members of the
association were issued non-member gate pass stickers
for their vehicles; a fact not disputed by SCHA. Thus, the
SCHA recognized that there were subdivision
landowners who were not members thereof,
notwithstanding the provisions of its Articles of
Incorporation and By-laws.

5.

Alliance of Concerned Government Workers v.


Minister of Labor and Employment

GUTIERREZ, JR., J.:


Are the branches, agencies, subdivisions, and
instrumentalities of the Government, including
government owned or controlled corporations included
among the 4 "employers"" under Presidential Decree No.
851 which are required to pay an their employees
receiving a basic salary of not more than P1,000.00 a
month, a thirteenth (13th) month pay not later than
December 24 of every year?
Petitioner Alliance of Government Workers (AGW) is a
registered labor federation while the other petitioners are
its affiliate unions with members from among the
employees of the following offices, schools, or
government owned or controlled corporations:
1. Philippine National Bank (PNB)
Escolta Street, Manila
2. Metropolitan Waterworks and
Sewerage System (MWSS) Katipunan
Road, Balara, Quezon City
3. Government Service Insurance
System (GSIS) Arroceros Street, Manila
4. Social Security System (SSS) East
Avenue, Quezon City
5. Philippine Virginia Tobacco
Administration (PVTA) Consolacion
Building, Cubao, Quezon City
6. Philippine Normal College (PNC)
Ayala Boulevard, Manila
7. Polytechnic University of the
Philippines (PUP) Hippodromo Street,
Sta. Mesa, Manila
On February 28, 1983, the Philippine Government
Employees Association (PGEA) filed a motion to come in
as an additional petitioner.
Presidential Decree No. 851 provides in its entirety:
WHEREAS, it is necessary to further
protect the level of real f wages from the
ravage of world-wide inflation;
WHEREAS, there has been no increase
case in the legal minimum wage rates
since 1970;
WHEREAS, the Christmas season is an
opportune time for society to show its

COMPILATION OF CASE DIGEST CONSTI 2: LIBERTY OF ABODE AND TRAVEL AND FREEDOM OF ASSOCIATION
ISV Notes

concern for the plight of the working


masses so they may properly celebrate
Christmas and New Year.
NOW, THEREFORE, I, FERDINAND E.
MARCOS, by virtue of the powers
vested in me by the Constitution do
hereby decree as follows:
SECTION 1. All employers are hereby
required to pay all their employees
receiving a basic salary of not more than
Pl,000 a month, regardless of the nature
of their employment, a 13th-month pay
not later than December 24 of every
year.
SECTION 2. Employers already paying
their employees a 13th-month pay or its
equivalent are not covered by this
Decree.
SECTION 3. This Decree shall take
effect immediately. Done in the City of
Manila, this 16th day of December 1975.
According to the petitioners, P.D. No. 851 requires all
employers to pay the 13th-month pay to their employees
with one sole exception found in Section 2 which states
that "(E)mployers already paying their employees a 13th
month pay or its equivalent are not covered by this
Decree. " The petitioners contend that Section 3 of the
Rules and Regulations Implementing Presidential
Decree No. 851 included other types of employers not
exempted by the decree. They state that nowhere in the
decree is the secretary, now Minister of Labor and
Employment, authorized to exempt other types of
employers from the requirement.
Section 3 of the Rules and Regulations Implementing
Presidential Decree No. 851 provides:
Section 3. Employers covered The
Decree shall apply to all employers
except to:
a) Distressed employers, such as (1)
those which are currently incurring
substantial losses or 112) in the case of
non-profit institutions and organizations,
where their income, whether from
donations, contributions, grants and
other earnings from any source, has
consistently declined by more than forty
(40%) per cent of their normal income
for the last two (2) )years, subject to the
provision of Section 7 of this issuance.

b) The Government and any of its


political subdivisions, including
government-owned and controlled
corporations, except)t those corporation,
operating essentially as private,
,subsidiaries of the government;
c) Employers already paying their
employees 13th-month pay or more in a
calendar year or its equivalent at the of
this issuance;
d) Employers of household helpers and
persons in the personal service of
another in relation to such workers: and
e) Employers of those who are paid on
purely commission, boundary, or task
basis and those who are paid a fixed for
performing a specific work, irrespective
of the time consumed in the
performance thereof, except where the
workers are paid an piece- rate basis in
which case the employer shall be
covered by this issuance :insofar ab
such workers are concerned ...
The petitioners assail this rule as ultra vires and
void. Citing Philippine Apparel Workers'Union v. NIRC et
al., (106 SCRA 444); Teoxon v. Members of the Board
of' Administators (33 SCRA 585); Santos u. Hon.
Estenzo et al., (109 Phil. 419); Hilado u. Collector of
Internal Revenue (100 Phil. 288), and Olsen & Co. Inc.
v. Aldanese and Trinidad (43 Phil. 259), the petitioners
argue that regulations adopted under legislative authority
must be in harmony with the provisions of the law and
for the sole purpose of carrying into effect its general
provisions. They state that a legislative act cannot be
amended by a rule and an administrative officer cannot
change the law. Section 3 is challenged as a substantial
modification by rule of a Presidential Decree and an
unlawful exercise of legislative power.
Our initial reaction was to deny due course to the petition
in a minute resolution, however, considering the
important issues propounded and the fact, that
constitutional principles are involved, we have now
decided to give due course to the petition, to consider
the various comments as answers and to resolve the
questions raised through a full length decision in the
exercise of this Court's symbolic function as an aspect of
the power of judicial review.
At the outset, the petitioners are faced with a procedural
barrier. The petition is one for declaratory relief, an
action not embraced within the original jurisdiction of the
Supreme Court. (Remotigue v. Osmena,, Jr., 21 SCRA
837; Rural Bank of Olongapo v. Commission of Land
Registration, 102 SCRA 794; De la Llana v. Alba, 112

COMPILATION OF CASE DIGEST CONSTI 2: LIBERTY OF ABODE AND TRAVEL AND FREEDOM OF ASSOCIATION
ISV Notes

SCRA 294). There is no statutory or jurisprudential basis


for the petitioners' statement that the Supreme Court has
original and exclusive jurisdiction over declaratory relief
suits where only questions of law are involved.
Jurisdiction is conferred by law. The petitioners have not
pointed to any provision of the Constitution or statute
which sustains their sweeping assertion. On this ground
alone, the petition could have been dismissed outright.
Following similar action taken in Nacionalista Party v.
Angelo Bautista (85 Phil. 101) and Aquino v.
Commission on Elections (62 SCRA 275) we have,
however, decided to treat the petition as one for
mandamus. The petition has far reaching implications
and raises questions that should be resolved. Have the
respondents unlawfully excluded the petitioners from the
use and enjoyment of rights to which they are entitled
under the law?
An analysis of the "whereases" of P.D. No. 851 shows
that the President had in mind only workers in private
employment when he issued the decree. There was no
intention to cover persons working in the government
service. The decree states:
xxx xxx xxx
WHEREAS, there has been no increase
in the legal minimum wage rates since
1970;
xxx xxx xxx
As pointed out by the Solicitor General in his comment
for the Minister of Labor and Employment, the Social
Security System the Philippine Normal College, and
Polytechnic University, the contention that govermment
owned and controlled corporations and state colleges
and universities are covered by the term "all employers"
is belied by the nature of the 13- month pay and the
intent behind the decree.
The Solicitor General states:
"Presidential Decree No. 851 is a labor standard law
which requires covered employers to pay their
employees receiving not more than P1,000.00 a month
an additional thirteenth-month pay. Its purpose is to
increase the real wage of the worker (Marcopper Mining
Corp. v. Ople, 105 SCRA 75; and National Federation of
Sugar Workers v. Ovejera, G.R. No. 59743, May 31,
1982) as explained in the'whereas'clause which read:
WHEREAS, it is
necessary to further
protect the level of real
wages from the ravage
of world-wide inflation;

WHEREAS, there has


been no increase in the
legal minimum wage
rates since 1970; 11
WHEREAS, the
Christmas season is an
opportune time for
society to show its
concern for the plight of
the working masses so
they may celebrate the
Christmas and New
Year.
xxx xxx xxx
What the P.D. No. 851 intended to
cover, as explained in the prefatory
statement of the Decree, are only those
in the private sector whose real wages
require protection from world-wide
inflation. This is emphasized by the
"whereas" clause which states that
'there has been no increase in the legal
minimum wage rates since 1970'. This
could only refer to the private sector,
and not to those in the government
service because at the time of the
enactment of Presidential Decree No.
851 in 1975, only the employees in the
private sector had not been given any
increase in their minimum wage. The
employees in the government service
had already been granted in 1974 a ten
percent across-the-board increase on
their salaries as stated in P.D. No. 525,
Section 4.
Moreover, where employees in the
government service were to benefit from
labor standard laws, their coverage is
explicitly stated in the statute or
presidential enactment. This is evident
in (a) Presidential Decree No. 390, Sec.
1 which granted emergency cost of
living allowance to employees in the
national government; (b) Republic Act
No. 6111, Sec. 10 on medicare benefits;
(c) Presidential Decree No -442, Title II,
Article 97 on the applicable minimum
wage rates; (d) Presidential Decree No.
442, Title 11, Article 167 (g) on
workmen's compensation; (e)
Presidential Decree No. 1123 which
provides for increases in emergency
allowance to employees in the private
sector and in salary to government
employees in Section 2 thereof; and (f)
Executive Order No. 752 granting

COMPILATION OF CASE DIGEST CONSTI 2: LIBERTY OF ABODE AND TRAVEL AND FREEDOM OF ASSOCIATION
ISV Notes

government employees a year-end


bonus equivalent to one week's pay.
Thus, had the intention been to include
government employees under the
coverage of Presidential Decree No.
851, said Decree should have expressly
so provided and there should have been
accompanying yearly appropriation
measures to implement the same. That
no such express provision was provided
and no accompanying appropriation
measure to was passed clearly show
the intent to exclude government
employees from the coverage of P. D.
No. 85 1.
We agree.
It is an old rule of statutory construction that restrictive
statutes and acts which impose burdens on the public
treasury or which diminish rights and interests, no matter
how broad their terms do not embrace the Sovereign,
unless the Sovereign is specifically mentioned. (See
Dollar Savings Bank v. United States, 19 Wall (U.S.)
227; United States v. United Mine Workers of America,
330 U.S. 265). The Republic of the Philippines, as
sovereign, cannot be covered by a general term like
"employer" unless the language used in the law is clear
and specific to that effect.
The issue raised in this petition, however, is more basic
and fundamental than a mere ascertainment of intent or
a construction of statutory provisions. It is concerned
with a revisiting of the traditional classification of
government employment into governmental functions
and proprietary functions and of the many ramifications
that this dichotomous treatment presents in the handling
of concerted activities, collective bargaining, and strikes
by government employees to wrest concessions in
compensation, fringe benefits, hiring and firing, and
other terms and conditions of employment.
The workers in the respondent institutions have not
directly petitioned the heads of their respective offices
nor their representatives in the Batasang Pambansa.
They have acted through a labor federation and its
affiliated unions. In other words, the workers and
employees of these state firms, college, and university
are taking collective action through a labor federation
which uses the bargaining power of organized labor to
secure increased compensation for its members.
Under the present state of the law and pursuant to the
express language of the Constitution, this resort to
concerted activity with the ever present threat of a strike
can no longer be allowed.
The general rule in the past and up to the present is that
"the terms and conditions of employment in the

Government, including any political subdivision or


instrumentality thereof are governed by law" (Section 11,
the Industrial Peace Act, R.A. No. 875, as amended and
Article 277, the Labor Code, P.D. No. 442, as amended).
Since the terms and conditions of government
employment are fixed by law, government workers
cannot use the same weapons employed by workers in
the private sector to secure concessions from their
employers. The principle behind labor unionism in
private industry is that industrial peace cannot be
secured through compulsion by law. Relations between
private employers and their employees rest on an
essentially voluntary basis. Subject to the minimum
requirements of wage laws and other labor and welfare
legislation, the terms and conditions of employment in
the unionized private sector are settled through the
process of collective bargaining. In government
employment, however, it is the legislature and, where
properly given delegated power, the administrative
heads of government which fix the terms and conditions
of employment. And this is effected through statutes or
administrative circulars, rules, and regulations, not
through collective bargaining agreements.
At the same time, the old Industrial Peace Act excepted
employees and workers in proprietary functions of
government from the above compulsion of law. Thus, in
the past, government employees performing proprietary
functions could belong to labor organizations imposing
the obligation to join in strikes or engage in other
concerted action. (Section 11, R.A. 875, as amended).
They could and they did engage in concerted activities
and various strikes against government owned and
controlled corporations and other government institutions
discharging proprietary functions. Among the institutions
as falling under the exception in Section 11 of the
Industrial Peace Act were respondents Government
Service Insurance System (GSISEA v. Alvendia, 108
Phil. 505) and Social Security System (SSSEA v.
Soriano, 7 SCRA 1016). And this Court has supported
labor completely in the various strikes and concerted
activities in firms and agencies discharging proprietary
functions because the Constitution and the laws allowed
these activities.
The exception, however belongs to the past.
The petitioners state in their counter comment filed July
23, 1982 that the 1973 Constitution is categorical about
the grant of the rights to self- organization and collective
bargaining to all workers and that no amount of
stretched interpretation of lesser laws like the Labor
Code and the Civil Service Act can overturn the clear
message of the Constitution with respect to these rights
to self-organization and collective bargaining.
These statements of the petitioners are error insofar as
government workers are now concerned.

COMPILATION OF CASE DIGEST CONSTI 2: LIBERTY OF ABODE AND TRAVEL AND FREEDOM OF ASSOCIATION
ISV Notes

Under the present Constitution, govemment-owned or


controlled corporations are specifically mentioned as
embraced by the civil service. (Section 1, Article XII-B,
Constitution). The inclusion of the clause "including
every government owned or controlled corporation" in
the 1973 amendments to the Constitution was a
deliberate amendment for an express purpose. There
may be those who disagree with the intent of the framers
of the amendment but because it is fundamental law, we
are all bound by it. The amendment was intended to
correct the situation where more favored employees of
the government could enjoy the benefits of two worlds.
They were protected by the laws governing government
employment. They could also engage in collective
bargaining and join in strikes to secure higher wages
and fringe benefits which equally hardworking
employees engaged in government functions could only
envy but not enjoy.
Presidential Decree No. 807, the Civil Service Decree of
the Philippines has implemented the 1973 Constitutional
amendment. It is categorical about the inclusion of
personnel of government-owned or controlled
corporations in the civil service and their being subject to
civil service requirements:
SECTION 56. Government- owned or
Controlled Corporations Personnel.All
permanent personnel of governmentowned or controlled corporations whose
positions are now embraced in the civil
service shall continue in the service until
they have been given a chance to
qualify in an appropriate examination,
but in the meantime, those who do not
possess the appropriate civil service
eligibility shall not be promoted until they
qualify in an appropriate civil service
examination. Services of temporary
personnel ma be y terminated any time.
Personnel of government-owned or controlled
corporations are now part of the civil service. It would not
be fair to allow them to engage in concerted activities to
wring higher salaries or fringe benefits from Government
even as other civil service personnel such as the
hundreds of thousands of public school teachers,
soldiers, policemen, health personnel, and other
government workers are denied the right to engage in
similar activities.
To say that the words "all employers" in P.D. No. 851
includes the Government and all its agencies,
instrumentalities, and government-owned or controlled
corporations would also result in nightmarish budgetary
problems.
For instance, the Supreme Court is trying its best to
alleviate the financial difficulties of courts, judges, and

court personnel in the entire country but it can do so only


within the limits of budgetary appropriations. Public
school teachers have been resorting to what was
formerly unthinkable, to mass leaves and
demonstrations, to get not a 13th-month pay but
promised increases in basic salaries and small
allowances for school uniforms. The budget of the
Ministry of Education, Culture and Sports has to be
supplemented every now and then for this purpose. The
point is, salaries and fringe benefits of those embraced
by the civil service are fixed by law. Any increases must
come from law, from appropriations or savings under the
law, and not from concerted activity.
The Government Corporate Counsel, Justice Manuel
Lazaro, in his consolidated comment * for respondents
GSIS, MWSS, and PVTA gives the background of the
amendment which includes every government-owned or
controlled corporation in the embrace of the civil service:
Records of the 1971 Constitutional
Convention show that in the
deliberations held relative to what is now
Section 1(1) Article XII-B, supra the
issue of the inclusion of governmentowned or controlled corporations figured
prominently.
The late delegate Roberto S. Oca, a
recognized labor leader, vehemently
objected to the inclusion of governmentowned or controlled corporations in the
Civil Service. He argued that such
inclusion would put asunder the right of
workers in government corporations,
recognized in jurisprudence under the
1935 Constitution, to form and join labor
unions for purposes of collective
bargaining with their employers in the
same manner as in the private section
(see: records of 1971 Constitutional
Convention).
In contrast, other labor experts and
delegates to the 1971 Constitutional
Convention enlightened the members of
the Committee on Labor on the
divergent situation of government
workers under the 1935 Constitution,
and called for its rectification. Thus, in a
Position Paper dated November-22,
1971, submitted to the Committee on
Labor, 1971 Constitutional Convention,
then Acting Commissioner of Civil
Service Epi Rev Pangramuyen
declared:
It is the stand, therefore,
of this Commission that

COMPILATION OF CASE DIGEST CONSTI 2: LIBERTY OF ABODE AND TRAVEL AND FREEDOM OF ASSOCIATION
ISV Notes

by reason of the nature


of the public employer
and the peculiar
character of the public
service, it must
necessarily regard the
right to strike given to
unions in private
industry as not applying
to public employees and
civil service employees.
It has been stated that
the Government, in
contrast to the private
employer, protects the
interests of all people in
the public service, and
that accordingly, such
conflicting interests as
are present in private
labor relations could not
exist in the relations
between government
and those whom they
employ.
Moreover, determination
of employment
conditions as well as
supervision of the
management of the
public service is in the
hands of legislative
bodies. It is further
emphasized that
government agencies in
the performance of their
duties have a right to
demand undivided
allegiance from their
workers and must
always maintain a
pronounced esprit de
corps or firm discipline
among their staff
members. It would be
highly incompatible with
these requirements of
the public service, if
personnel took orders
from union leaders or
put solidarity with
members of the working
class above solidarity
with the Government.
This would be inimical
to the public interest.
Moreover, it is asserted
that public employees

by joining labor unions


may be compelled to
support objectives
which are political in
nature and thus
jeopardize the
fundamental principle
that the governmental
machinery must be
impartial and nonpolitical in the sense of
party politics.' (see:
Records of 1971
Constitutional
Convention).
Similarly, Delegate Leandro P. Garcia,
expressing support for the inclusion of
government-owned or controlled
corporations in the Civil Service, argued:
It is meretricious to
contend that because
Govermnent owned or
controlled corporations
yield profits, their
employees are entitled
to better wages and
fringe benefits than
employees of
Government other than
Government- owned
and controlled cor
orations which are not
making profits. There is
no gainsaying the fact
that the capital they use
is the people's (see
Records of the 1971
Constitutional
Convention).
Summarizing the deliberations of the
1971 Constitutional Convention on the
inclusion of Government owned or
controlled corporations, Dean Joaquin
G. Bernas, SJ., of the Ateneo de Manila
University Professional School of Law,
stated that government-owned
corporations came under attack as
milking cows of a privileged few
enjoying salaries far higher than their
counterparts in the various branches of
government, while the capital of these
corporations belongs to the Government
and government money is pumped into
them whenever on the brink of disaster,
and they should therefore come under
the strick surveillance of the Civil
Service System. (Bernas, The 1973

COMPILATION OF CASE DIGEST CONSTI 2: LIBERTY OF ABODE AND TRAVEL AND FREEDOM OF ASSOCIATION
ISV Notes

Philippine Constitution, Notes and


Cases, 1974 ed., p. 524).
The Government Corporate Counsel cites the precedent
setting decision in Agricultural- Credit and Cooperative
Financing Administration (ACCFA v. Confederation of
Unions in Government Corporations and Offtces
CUGCO et al., 30 SCRA 649) as giving the rationale for
coverage of government-owned or controlled
corporations by the civil service. We stated ACCFA v.
CUGCO that:
... The ACA is a government office or
agency engaged in governmental, not
proprietary functions. These functions
may not be strictly what President
Wilson described as "constituent" (as
distinguished from 'ministrant'), [Bacani
vs. National Coconut Corporation, G.R.
No. L-9657, Nov. 29,1956, 53 O.G. p.
2800] such as those relating to the
maintenance of peace and the
prevention of crime, those regulating
property and property rights, those
relating to the administration of justice
and the determination of political duties
of citizens, and those relating to national
defense and foreign relations. Under
this traditional classification, such
constituent functions are exercised by
the State as attributes of sovereignty,
and not merely to promote the welfare,
progress and prosperity of the people
these latter functions being ministrant,
the exercise of which is optional on the
part of the government.
The growing complexities of modern
society, however, have rendered this
traditional classification of the functions
of government quite unrealistic, not to
say obsolete. The areas which used to
be left to private enterprise and initiative
and which the government was called
upon to enter optionally, and only
"because it was better equipped to
administer for the public welfare than is
any private individual or group of
individuals," (Malcolm, The Government
of the Philippines, pp. 19-20; Bacani vs.
National Coconut Corporation, supra)
continue to lose their well- defined
boundaries and to be absorbed within
activities that the government must
undertake in its sovereign capacity if it is
to meet the increasing social challenges
of the times. Here as almost everywhere
else the tendency is undoubtedly
towards a greater socialization of
economic forces, Here of course this

development was envisioned, indeed


adopted as a national policy, by the
Constitution itself in its declaration of
principle concerning the promotion of
social justice.
Chief Justice Fernando, then an Associate Justice of this
Court, observed in a concurring opinion that the
traditional classification into constituent and ministrant
functions reflects the primacy at that time of the now
discredited and repudiated laissez faire concept carried
over into government. He stated:
The influence exerted by American
constitutional doctrines unavoidable
when the Philippines was still under
American rule notwithstanding, an
influence that has not altogether
vanished even after independence,
the laissez faire principle never found
fun acceptance in this jurisdiction, even
during the period of its full flowering in
the United States. Moreover, to erase
any doubts, the Constitutional
Convention saw to it that our
fundamental law embodies a policy of
the responsibility thrust on government
to cope with social and economic
problems and an earnest and sincere
commitment to the promotion of the
general welfare through state action. It
would thus follow that the force of any
legal objection to regulatory measures
adversely affecting property rights or to
statutes organizing public corporations
that may engage in competition with
private enterprise has been blunted.
Unless there be a clear showing of any
invasion of rights guaranteed by the
Constitution, their validity is a foregone
conclusion. No fear need be entertained
that thereby spheres hitherto deemed
outside government domain have been
encroached upon. With our explicit
disavowal of the 'constituent-ministrant'
test, the ghost of the laissez-faire
concept no longer stalks the juridical
stage."
Our dismissal of this petiti/n should not, by any means,
be interpreted to imply that workers in governmentowned and controlled corporations or in state colleges
and universities may not enjoy freedom of association.
The workers whom the petitioners purport to represent
have the right, which may not be abridged, to form
associations or societies for purposes not contrary to
law. (Constitution, Article IV, Section 7). This is a right
which share with all public officers and employees and,
in fact, by everybody living in this country. But they may
not join associations which impose the obligation to

COMPILATION OF CASE DIGEST CONSTI 2: LIBERTY OF ABODE AND TRAVEL AND FREEDOM OF ASSOCIATION
ISV Notes

engage in concerted activities in order to get salaries,


fringe benefits, and other emoluments higher than or
different frm that provided by law and regulation.
The very Labor Code, P.D. No. 442 as amended,, which
governs the registration and provides for the rights of
legitimate labor organizations states:
ART. 277. Government employees.
The terms and conditions of
employment of all government
employees, including employees of
government-owned and controlled
corporations, shall be governed by the
Civil Service Law, rules and regulations.
Their salaries shall be standardized by
the National Assembly as provided for in
the new constitution. However, there
shall be no reduction of existing wages,
benefits, and other terms and conditions
of employment being enjoyed by them
at the time of the adoption of this code.
Section 6, Article XII-B of the Constitution gives added
reasons why the government employees represented by
the petitioners cannot expect treatment in matters of
salaries different from that extended to all others
government personnel. The provision states:
SEC. 6. The National Assembly shall
provide for the standardization of
compensation of government officials
and employees, including those in
government-owned or controlled
corporations, taking into account the
nature of the responsibilities pertaining
to, and the qualifications required for the
positions concerned.
It is the legislature or, in proper cases, the administrative
heads of government and not the collective bargaining
process nor the concessions wrung by labor unions from
management that determine how much the workers in
government-owned or controlled corporations may
receive in terms of salaries, 13th month pay, and other
conditions or terms of employment. There are
government institutions which can afford to pay two
weeks, three weeks, or even 13th-month salaries to their
personnel from their budgetary appropriations. However,
these payments must be pursuant to law or regulation.
Presidential Decree No. 985 as amended provides:
xxx xxx xxx
SEC. 2. Declaration of Policy. It is
hereby declared to be the policy, of the
national government to provide equal
pay for substantially, equal work and to
base differences in pay upon

substantive differences in duties and


responsibilities, and qualification
requirements of the positions. In
determining rates of pay, due regard
shall be given to, among others,
prevailing rates in private industry for
comparable work. For this purpose,
there is hereby established a system of
compensation standardization and
position classification in the national
government for all departments,
bureaus, agencies, and officers
including government-owned or
controlled corporations and financial
institutions: Provided, That
notwithstanding a standardized salary
system established for all employees,
additional financial incentives may be
established by government corporations
and financial institutions for their
employees to be supported fully from
their corporate funds and for such
technical positions as may be approved
by the President in critical government
agencies.
The Solicitor-General correctly points out that to interpret
P.D. No. 851 as including government employees would
upset the compensation levels of government
employees in violation of those fixed according to P.D.
No. 985.
Here as in other countries, government salaries and
wages have always been lower than salaries, wages,
and bonuses in the private sector. However, civil
servants have no cause for despair. Service in the
government may at times be a sacrifice but it is also a
welcome privilege. Apart from the emotional and psychic
satisfactions, there are various material advantages. The
security of tenure guaranteed to those in the civil service
by the Constitution and statutes, the knowledge that one
is working for the most stable of employers and not for
private persons, the merit system in appointments and
promotions, the scheme of vacation, sick, and maternity
leave privileges, and the prestige and dignity associated
with public office are only a few of the joys of
government employment.
Section 3 of the Rules and Regulations Implementing
Presidential Decree No. 851 is, therefore, a correct
interpretation of the decree. It has been implemented
and enforced from December 22, 1975 to the present,

The petitioners have shown no valid reason why it


should be nullified because of their petition filed six
and a half years after the issuance and
implementation of the rule.
WHEREFORE, the petition is hereby DISMISSED for
lack of merit.

COMPILATION OF CASE DIGEST CONSTI 2: LIBERTY OF ABODE AND TRAVEL AND FREEDOM OF ASSOCIATION
ISV Notes

SO ORDERED.

6. GSIS vs Mga Kapisanan ng mga Manggagawa


sa GSIS
In this petition for review on certiorari under
Rule 45 of the Rules of Court, the Government Service
Insurance System (GSIS) and its President and General
Manager Winston F. Garcia (Garcia, for short) assail and
seek to nullify the Decision1 dated June 16, 2005 of the
Court of Appeals (CA) in CA-G.R. SP No. 87220, as
reiterated in its Resolution2 of October 18, 2005 denying
Garcia's motion for reconsideration.
The recourse is cast against the following setting:
A four-day October 2004 concerted demonstration,
rallies and en masse walkout waged/held in front of the
GSIS main office in Roxas Boulevard, Pasay City,
started it all. Forming a huge part of the October 4 to
October 7, 2004 mass action participants were GSIS
personnel, among them members of the herein
respondent Kapisanan Ng Mga Manggagawa sa GSIS
("KMG" or the "Union"), a public sector union of GSIS
rank-and-file employees. Contingents from other
government agencies joined causes with the GSIS
group. The mass action's target appeared to have been
herein petitioner Garcia and his management style.
While the Mayor of Pasay City allegedly issued a rally
permit, the absence of the participating GSIS employees
was not covered by a prior approved leave.3
On or about October 10, 2004, the manager of the GSIS
Investigating Unit issued a memorandum directing 131
union and non-union members to show cause why they
should not be charged administratively for their
participation in said rally. In reaction, KMG's counsel,
Atty. Manuel Molina, sought reconsideration of said
directive on the ground, among others, that the subject
employees resumed work on October 8, 2004 in
obedience to the return-to-work order thus issued. The
plea for reconsideration was, however, effectively denied
by the filing, on October 25, 2004, of administrative
charges against some 110 KMG members for grave
misconduct and conduct prejudicial to the best interest of
the service.4
What happened next is summarized by the CA in its
challenged decision of June 16, 2005, albeit the herein
petitioners would except from some of the details of the
appellate court's narration:

Ignoring said formal charges, KMG, thru its


President, Albert Velasco, commenced the
instant suit on November 2, 2004, with the filing
of the Petition for Prohibition at bench. On the
ground that its members should not be made to
explain why they supported their union's cause,
petitioner [KMG] faulted respondent [Garcia]
with blatant disregard of Civil Service Resolution
No. 021316, otherwise known as the Guidelines
for Prohibited Mass Action, Section 10 of which
exhorts government agencies to "harness all
means within their capacity to accord due regard
and attention to employees' grievances and
facilitate their speedy and amicable disposition
through the use of grievance machinery or any
other modes of settlement sanctioned by law
and existing civil service rules." Two
supplements to the foregoing petition were
eventually filed by KMG. The first, apprised
[the CA] of the supposed fact that its Speaker,
Atty. Molina, had been placed under preventive
suspension for 90 days and that the formal
charges thus filed will not only deprive its
members of the privileges and benefits due
them but will also disqualify them from
promotion, step increment adjustments and
receipt of monetary benefits, including their 13th
month pay and Christmas bonuses. The second,
xxx manifested that, on December 17, 2004,
respondent [Garcia] served a spate of additional
formal charges against 230 of KMG's members
for their participation in the aforesaid grievance
demonstrations.
In his December 14, 2004 comment to the
foregoing petition, respondent [Garcia] averred
that the case at bench was filed by an
unauthorized representative in view of the fact
that Albert Velasco had already been dropped
from the GSIS rolls and, by said token, had
ceased to be a member much less the
President of KMG. Invoking the rule against
forum shopping, respondent [Garcia] called [the
CA's] attention to the supposed fact that the
allegations in the subject petition merely
duplicated those already set forth in two
petitions for certiorari and prohibition earlier filed
by Albert Velasco . Because said petitions are,
in point of fact, pending before this court as CAG.R. SP Nos. 86130 and 86365, respondent
[Garcia] prayed for the dismissal of the petition
at bench .5 (Words in bracket added.)

COMPILATION OF CASE DIGEST CONSTI 2: LIBERTY OF ABODE AND TRAVEL AND FREEDOM OF ASSOCIATION
ISV Notes

It appears that pending resolution by the CA of the KMG


petition for prohibition in this case, the GSIS
management proceeded with the investigation of the
administrative cases filed. As represented in a pleading
before the CA, as of May 18, 2005, two hundred seven
(207) out of the two hundred seventy eight (278) cases
filed had been resolved, resulting in the exoneration of
twenty (20) respondent-employees, the reprimand of one
hundred eighty two (182) and the suspension for one
month of five (5).6

involving self-organization, strikes, demonstrations and


like concerted actions. In fact, policies established on
public sector unionism and rules issued on mass action
have been noted and cited by the Court in at least a
case.11 Among these issuances is Executive Order (EO)
No. 180, series of 1987, providing guidelines for the
exercise of the right to organize of government
employees. Relevant also is CSC Resolution No.
021316 which provides rules on prohibited concerted
mass actions in the public sector.

On June 16, 2005, the CA rendered the herein assailed


decision7 holding that Garcia's "filing of administrative
charges against 361 of [KMG's] members is tantamount
to grave abuse of discretion which may be the proper
subject of the writ of prohibition." Dispositively, the
decision reads:

There is hardly any dispute about the formal charges


against the 278 affected GSIS employees a mix of
KMG union and non-union members - having arose from
their having gone on unauthorized leave of absence
(AWOL) for at least a day or two in the October 4 to 7,
2004 stretch to join the ranks of the demonstrators
/rallyists at that time. As stated in each of the formal
charges, the employee's act of attending, joining,
participating and taking part in the strike/rally is a
transgression of the rules on strike in the public sector.
The question that immediately comes to the fore,
therefore, is whether or not the mass action staged by or
participated in by said GSIS employees partook of a
strike or prohibited concerted mass action. If in the
affirmative, then the denounced filing of the
administrative charges would be prima facie tenable,
inasmuch as engaging in mass actions resulting in work
stoppage or service disruption constitutes, in the
minimum, the punishable offense of acting prejudicial to
the best interest of the service.12 If in the negative, then
such filing would indeed smack of arbitrariness and
justify the issuance of a corrective or preventive writ.

WHEREFORE, premises considered, the


petition [of KMG] is GRANTED and respondent
[Winston F. Garcia] is
hereby PERPETUALLY ENJOINED from
implementing the issued formal charges and
from issuing other formal charges arising from
the same facts and events.
SO ORDERED. (Emphasis in the original)
Unable to accept the above ruling and the purported
speculative factual and erroneous legal premises holding
it together, petitioner Garcia sought reconsideration. In
its equally assailed Resolution8 of October 18, 2005,
however, the appellate court denied reconsideration of
its decision.
Hence, this recourse by the petitioners ascribing serious
errors on the appellate court in granting the petition for
prohibition absent an instance of grave abuse of
authority on their part.
We resolve to GRANT the petition.
It should be stressed right off that the civil service
encompasses all branches and agencies of the
Government, including government-owned or controlled
corporations (GOCCs) with original charters, like the
GSIS,9 or those created by special law.10 As such,
employees of covered GOCCs are part of the civil
service system and are subject to circulars, rules and
regulations issued by the Civil Service Commission
(CSC) on discipline, attendance and general
terms/conditions of employment, inclusive of matters

Petitioners assert that the filing of the formal charges are


but a natural consequence of the service-disrupting
rallies and demonstrations staged during office hours by
the absenting GSIS employees, there being appropriate
issuances outlawing such kinds of mass action. On the
other hand, the CA, agreeing with the respondent's
argument, assumed the view and held that the organized
demonstrating employees did nothing more than air their
grievances in the exercise of their "broader rights of free
expression"13 and are, therefore, not amenable to
administrative sanctions. For perspective, following is
what the CA said:
Although the filing of administrative charges
against [respondent KMG's] members is well
within [petitioner Garcia's] official [disciplinary]
prerogatives, [his] exercise of the power vested
under Section 45 of Republic Act No. 8291 was
tainted with arbitrariness and vindictiveness

COMPILATION OF CASE DIGEST CONSTI 2: LIBERTY OF ABODE AND TRAVEL AND FREEDOM OF ASSOCIATION
ISV Notes

against which prohibition was sought by


[respondent]. xxx the fact that the subject mass
demonstrations were directed against [Garcia's]
supposed mismanagement of the financial
resources of the GSIS, by and of itself, renders
the filing of administrative charges against
[KMG's] member suspect. More significantly, we
find the gravity of the offenses and the sheer
number of persons charged administratively
to be, at the very least, antithetical to the best
interest of the service.
It matters little that, instead of the 361 alleged by
petitioner, only 278 charges were actually filed
[and] in the meantime, disposed of and of the
said number, 20 resulted to exoneration, 182 to
reprimand and 5 to the imposition of a penalty of
one month suspension. Irrespective of their
outcome, the severe penalties prescribed for the
offense with which petitioner's members were
charged, to our mind, bespeak of bellicose and
castigatory reaction . The fact that most of the
employees [Garcia] administratively charged
were eventually meted with what appears to be
a virtual slap on the wrist even makes us wonder
why respondent even bothered to file said
charges at all. xxx.
Alongside the consequences of the right of
government employees to form, join or assist
employees organization, we have already
mentioned how the broader rights of free
expression cast its long shadow over the case.
xxx we find [petitioner Garcia's] assailed acts, on
the whole, anathema to said right which has
been aptly characterized as preferred, one
which stands on a higher level than substantive
economic and other liberties, the matrix of other
important rights of our people.
xxx.14 (Underscoring and words in bracket
added; citations omitted.)
While its decision and resolution do not explicitly say so,
the CA equated the right to form associations with the
right to engage in strike and similar activities available to
workers in the private sector. In the concrete, the
appellate court concluded that inasmuch as GSIS
employees are not barred from forming, joining or
assisting employees' organization, petitioner Garcia
could not validly initiate charges against GSIS
employees waging or joining rallies and demonstrations
notwithstanding the service-disruptive effect of such
mass action. Citing what Justice Isagani Cruz said

in Manila Public School Teachers Association [MPSTA]


v. Laguio, Jr.,15 the appellate court declared:
It is already evident from the aforesaid
provisions of Resolution No. 021316 that
employees of the GSIS are not among those
specifically barred from forming, joining or
assisting employees organization such as
[KMG]. If only for this ineluctable fact, the merit
of the petition at bench is readily discernible.16
We are unable to lend concurrence to the above CA
posture. For, let alone the fact that it ignores what the
Court has uniformly held all along, the appellate court's
position is contrary to what Section 4 in relation to
Section 5 of CSC Resolution No. 02131617 provides.
Besides, the appellate court's invocation of Justice
Cruz's opinion inMPSTA is clearly off-tangent, the good
Justice's opinion thereat being a dissent. It may be, as
the appellate court urged that the freedom of expression
and assembly and the right to petition the government
for a redress of grievances stand on a level higher than
economic and other liberties. Any suggestion, however,
about these rights as including the right on the part of
government personnel to strike ought to be, as it has
been, trashed. We have made this abundantly clear in
our past determinations. For instance, in Alliance of
Government Workers v. Minister of Labor and
Employment,18 a case decided under the aegis of the
1973 Constitution, an en banc Court declared that it
would be unfair to allow employees of government
corporations to resort to concerted activity with the ever
present threat of a strike to wring benefits from
Government. Then came the 1987 Constitution
expressly guaranteeing, for the first time, the right of
government personnel to self-organization19 to
complement the provision according workers the right to
engage in "peaceful concerted activities, including the
right to strike in accordance with law."20
It was against the backdrop of the aforesaid provisions
of the 1987 Constitution that the Court
resolvedBangalisan v. Court of Appeals.21 In it, we held,
citing MPSTA v. Laguio, Jr.,22 that employees in the
public service may not engage in strikes or in concerted
and unauthorized stoppage of work; that the right of
government employees to organize is limited to the
formation of unions or associations, without including the
right to strike.
Jacinto v. Court of Appeals23 came next and there we
explained:

COMPILATION OF CASE DIGEST CONSTI 2: LIBERTY OF ABODE AND TRAVEL AND FREEDOM OF ASSOCIATION
ISV Notes

Specifically, the right of civil servants to organize


themselves was positively recognized in
Association of Court of Appeals Employees vs.
Ferrer-Caleja. But, as in the exercise of the
rights of free expression and of assembly, there
are standards for allowable limitations such
as the legitimacy of the purpose of the
association, [and] the overriding considerations
of national security . . . .
As regards the right to strike, the Constitution
itself qualifies its exercise with the provision "in
accordance with law." This is a clear
manifestation that the state may, by law, regulate
the use of this right, or even deny certain sectors
such right. Executive Order 180 which provides
guidelines for the exercise of the right of
government workers to organize, for instance,
implicitly endorsed an earlier CSC circular which
"enjoins under pain of administrative sanctions,
all government officers and employees from
staging strikes, demonstrations, mass leaves,
walkouts and other forms of mass action which
will result in temporary stoppage or disruption of
public service" by stating that the Civil Service
law and rules governing concerted activities and
strikes in government service shall be observed.
(Emphasis and words in bracket added; citations
omitted)
And in the fairly recent case of Gesite v. Court of
Appeals,24 the Court defined the limits of the right of
government employees to organize in the following wise:
It is relevant to state at this point that the settled
rule in this jurisdiction is that employees in the
public service may not engage in strikes, mass
leaves, walkouts, and other forms of mass
action that will lead in the temporary stoppage or
disruption of public service. The right of
government employees to organize is limited to
the formation of unions or associations only,
without including the right to strike,
adding that public employees going on disruptive
unauthorized absences to join concerted mass actions
may be held liable for conduct prejudicial to the best
interest of the service.

Significantly, 1986 Constitutional Commission member


Eulogio Lerum, answering in the negative the poser of
whether or not the right of government employees to
self-organization also includes the right to strike, stated:
When we proposed this amendment providing
for self organization of government employees, it
does not mean that because they have the right
to organize, they have also the right to strike.
That is a different matter. xxx25
With the view we take of the events that transpired on
October 4-7, 2004, what respondent's members
launched or participated in during that time partook of a
strike or, what contextually amounts to the same thing, a
prohibited concerted activity. The phrase "prohibited
concerted activity" refers to any collective activity
undertaken by government employees, by themselves or
through their employees' organization, with the intent of
effecting work stoppage or service disruption in order to
realize their demands or force concessions, economic or
otherwise; it includes mass leaves, walkouts, pickets and
acts of similar nature.26 Indeed, for four straight days,
participating KMG members and other GSIS employees
staged a walk out and waged or participated in a mass
protest or demonstration right at the very doorstep of the
GSIS main office building. The record of attendance 27 for
the period material shows that, on the first day of the
protest, 851 employees, or forty eight per cent (48%) of
the total number of employees in the main office (1,756)
took to the streets during office hours, from 6 a.m. to 2
p.m.,28leaving the other employees to fend for
themselves in an office where a host of transactions take
place every business day. On the second day, 707
employees left their respective work stations, while 538
participated in the mass action on the third day. A
smaller number, i.e., 306 employees, but by no means
an insignificant few, joined the fourth day activity.
To say that there was no work disruption or that the
delivery of services remained at the usual level of
efficiency at the GSIS main office during those four (4)
days of massive walkouts and wholesale absences
would be to understate things. And to place the erring
employees beyond the reach of administrative
accountability would be to trivialize the civil service rules,
not to mention the compelling spirit of professionalism
exacted of civil servants by the Code of Conduct and
Ethical Standards for Public Officials and Employees. 29
The appellate court made specific reference to the
"parliament of the streets," obviously to lend
concurrence to respondent's pretension that the

COMPILATION OF CASE DIGEST CONSTI 2: LIBERTY OF ABODE AND TRAVEL AND FREEDOM OF ASSOCIATION
ISV Notes

gathering of GSIS employees on October 4-7, 2004 was


an "assembly of citizens" out only to air grievances, not
a striking crowd. According to the respondent, a strike
presupposes a mass action undertaken to press for
some economic demands or secure additional material
employment benefits.
We are not convinced.
In whatever name respondent desires to call the fourday mass action in October 2004, the stubborn fact
remains that the erring employees, instead of exploring
non-crippling activities during their free time, had taken a
disruptive approach to attain whatever it was they were
specifically after. As events evolved, they assembled in
front of the GSIS main office building during office hours
and staged rallies and protests, and even tried to
convince others to join their cause, thus provoking work
stoppage and service-delivery disruption, the very evil
sought to be forestalled by the prohibition against strikes
by government personnel.30
The Court can concede hypothetically that the protest
rally and gathering in question did not involve some
specific material demand. But then the absence of such
economic-related demand, even if true, did not, under
the premises, make such mass action less of a
prohibited concerted activity. For, as articulated earlier,
any collective activity undertaken by government
employees with the intent of effecting work stoppage or
service disruption in order to realize their demands or
force concessions, economic or otherwise, is a
prohibited concerted mass action31 and doubtless
actionable administratively. Bangalisan even went further
to say the following: "[i]n the absence of statute, public
employees do not have the right to engage in concerted
work stoppages for any purpose."
To petitioner Garcia, as President and General Manager
of GSIS, rests the authority and responsibility, under
Section 45 of Republic Act No. 8291, the GSIS Act of
1997, to remove, suspend or otherwise discipline GSIS
personnel for cause.32 At bottom then, petitioner Garcia,
by filing or causing the filing of administrative charges
against the absenting participants of the October 4-7,
2004 mass action, merely performed a duty expected of
him and enjoined by law. Regardless of the mood
petitioner Garcia was in when he signed the charge
sheet, his act can easily be sustained as legally correct
and doubtless within his jurisdiction.

It bears to reiterate at this point that the GSIS employees


concerned were proceeded against - and eventually
either exonerated, reprimanded or meted a one-month
suspension, as the case may be - not for the exercise of
their right to assemble peacefully and to petition for
redress of grievance, but for engaging in what appeared
to be a prohibited concerted activity. Respondent no less
admitted that its members and other GSIS employees
might have disrupted public service.33
To be sure, arbitrariness and whimsical exercise of
power or, in fine, grave abuse of discretion on the part of
petitioner Garcia cannot be simplistically inferred from
the sheer number of those charged as well as the gravity
or the dire consequences of the charge of grave
misconduct and conduct prejudicial to the best interest of
the service, as the appellate court made it to appear.
The principle of accountability demands that every erring
government employee be made answerable for any
malfeasance or misfeasance committed. And lest it be
overlooked, the mere filing of formal administrative case,
regardless of the gravity of the offense charged, does
not overcome the presumptive innocence of the persons
complained of nor does it shift the burden of evidence to
prove guilt of an administrative offense from the
complainant.
Moreover, the Court invites attention to its holding
in MPSTA v. Laguio, Jr., a case involving over 800 public
school teachers who took part in mass actions for which
the then Secretary of Education filed administrative
complaints on assorted charges, such as gross
misconduct. Of those charged, 650 were dismissed and
195 suspended for at least six (6) months The Court,
however, did not consider the element of number of
respondents thereat and/or the dire consequences of the
charge/s as fatally vitiating or beclouding the bona
fidesof the Secretary of Education's challenged action.
Then as now, the Court finds the filing of charges against
a large number of persons and/or the likelihood that they
will be suspended or, worse, dismissed from the service
for the offense as indicating a strong and clear case of
grave abuse of authority to justify the issuance of a writ
of prohibition.
The appellate court faulted petitioner Garcia for not first
taping existing grievance machinery and other modes of
settlement agreed upon in the GSIS-KMG Collective
Negotiations Agreement (CAN) before going full steam
ahead with his formal charges.34
The Court can plausibly accord cogency to the CA's
angle on grievance procedure but for the fact that it

COMPILATION OF CASE DIGEST CONSTI 2: LIBERTY OF ABODE AND TRAVEL AND FREEDOM OF ASSOCIATION
ISV Notes

conveniently disregarded what appears to be the more


relevant provision of the CNA. We refer to Article VI
which reads:
The GSIS Management and the KMG have
mutually agreed to promote the principle of
shared responsibility on all matters and
decisions affecting the rights, benefits and
interests of all GSIS employees . Accordingly,
the parties also mutually agree that the KMG
shall not declare a strike nor stage any
concerted action which will disrupt public service
and the GSIS management shall not
lockoutemployees who are members of the
KMG during the term of this agreement. GSIS
Management shall also respect the rights of the
employees to air their sentiments through
peaceful concerted activities during allowable
hours, subject to reasonable office
rules ....35 (Underscoring added)
If the finger of blame, therefore, is to be pointed at
someone for non-exhaustion of less confrontational
remedies, it should be at the respondent union for
spearheading a concerted mass action without resorting
to available settlement mechanism. As it were, it was
KMG, under Atty. Alberto Velasco, which opened fire
first. That none of the parties bothered to avail of the
grievance procedures under the GSIS-KMG CNA should
not be taken against the GSIS. At best, both GSIS
management and the Union should be considered as
in pari delicto.
With the foregoing disquisitions, the Court finds it
unnecessary to discuss at length the legal standing of
Alberto Velasco to represent the herein respondent
union and to initiate the underlying petition for
prohibition. Suffice it to state that Velasco, per Joint
Resolution No. 04-10-01 approved on October 5, 2004
by the KMG Joint Executive-Legislative Assembly, had
ceased to be member, let alone president, of the KMG,
having previously been dropped from the rolls of GSIS
employees.36 While the dropping from the rolls is alleged
to have been the subject of a CA-issued temporary
restraining order (TRO), the injunction came after Atty.
Velasco had in fact been separated from the service and
it appears that the TRO had already expired.
As a final consideration, the Court notes or reiterates the
following relevant incidents surrounding the disposition
of the case below:

1. The CA had invoked as part of its ratio


decidendi a dissenting opinion in MPSTA, even
going to the extent of describing as "instructive
and timely" a portion, when the majority opinion
thereat, which the appellate court ignored, is the
controlling jurisprudence.
2. The CA gave prominence to dispositions and
rattled off holdings37 of the Court, which
appropriately apply only to strikes in the private
industry labor sector, and utilized the same as
springboard to justify an inference of grave
abuse of discretion. On the other hand, it only
gave perfunctory treatment if not totally ignored
jurisprudence that squarely dealt with strikes in
the public sector, as if the right to strike given to
unions in private corporations/entities is
necessarily applicable to civil service
employees.
3. As couched, the assailed CA decision
perpetually bars respondent Garcia and
necessarily whoever succeeds him as GSIS
President not only from implementing the
formal charges against GSIS employees who
participated in the October 4 - 7, 2004 mass
action but also from issuing other formal charges
arising from the same events. The injunction
was predicated on a finding that grave abuse of
discretion attended the exercise of petitioner
Garcia's disciplinary power vested him under
Section 45 of RA 8291.38 At bottom then, the
assailed decision struck down as a nullity, owing
to the alleged attendant arbitrariness, not only
acts that have already been done, but those yet
to be done. In net effect, any formal charge
arising from the October 4-7, 2004 incident is,
under any and all circumstances, prejudged as
necessarily tainted with arbitrariness to be slain
at sight.
The absurdities and ironies easily deducible from the
foregoing situations are not lost on the Court.
We close with the observation that the assailed decision
and resolution, if allowed to remain undisturbed, would
likely pave the way to the legitimization of mass actions
undertaken by civil servants, regardless of their
deleterious effects on the interest of the public they have
sworn to serve with loyalty and efficiency. Worse still, it
would permit the emergence of a system where public
sector workers are, as the petitioners aptly put it,
"immune from the minimum reckoning for acts that

COMPILATION OF CASE DIGEST CONSTI 2: LIBERTY OF ABODE AND TRAVEL AND FREEDOM OF ASSOCIATION
ISV Notes

[under settled jurisprudence] are concededly unlawful."


This aberration would be intolerable.
WHEREFORE, the assailed Decision and Resolution of
the Court of Appeals are REVERSED and SET
ASIDE and the writ of prohibition issued by that court
is NULLIFIED.
No Cost.

SO ORDERED.
Puno, J., Chairperson, Sandoval-Gutierrez, and Azcuna,
JJ., concur.
Corona, J., On Leave.

Вам также может понравиться